Regel von de l'Hospital 
 
3HTAM Mathe-Hilfe online

Regel von de l'Hospital


Ein häufiges Thema in der Analysis, ist das Betrachten von Grenzwerten. Haben wir zum Beispiel die Funktion \[ f(x)=\frac{1}{x}\] so können wir vier besondere Grenzwerte betrachten. \begin{align} &\color{black}{\lim\limits_{x \to - \infty}f(x)}&&\color{green}{\lim\limits_{x \to + \infty}f(x)} \\ &\color{blue}{\lim\limits_{x \to 0^- }f(x)}&&\color{red}{\lim\limits_{x \to 0^+ }f(x)} \end{align}
3HTAM: Regel von de l'Hospital
Mit $\color{blue}{\lim\limits_{x \to 0^- }}$ ist der linksseitige Limes bei $x=0$ gemeint. In diesem Fall ist die Grenzwertbetrachtung allerdings sehr einfach. \begin{align} &\lim\limits_{x \to - \infty}f(x)=0&&\lim\limits_{x \to + \infty}f(x)=0 \\ &\lim\limits_{x \to 0^- }f(x)=-\infty&&\lim\limits_{x \to 0^+ }f(x)=+\infty \end{align} Schwerer wird es zum Beispiel bei: \[ \lim\limits_{x \to + \infty} \frac{e^x}{x}\] Häufig wird hier gesagt, dass $+ \infty$ als Grenzwert herauskommt, da die Exponentialfunktion dominiert. Ich möchte hier keine Schulrevolution betreiben, aber eine genauere Begründung geben, oder besser gesagt, ein Verfahren erklären, womit man viele neue Grenzwerte komplizierter Funktionen berechnet.

Fangen wir erst einmal mit dem Satz und einem Beispiel an. Anschließend kommt noch ein Beweis eines der Fälle.

Regeln von de l'Hospital

Es seien $a$, $b \in \mathbb{R} \cup\{+\infty, - \infty\}$. Die beiden Funktionen $f,g$ seien definiert auf dem Intervall $(a,b)$ und beide jeweils differenzierbar. Außerdem sei $g'(x) \ne 0$ für alle $x$. Trifft nun einer der beiden folgenden Aussagen
  1. $\lim\limits_{x \to a}f(x) = \lim\limits_{x \to a}g(x) =0$ oder
  2. $\lim\limits_{x \to a}f(x) = \pm \infty$ und $\lim\limits_{x \to a}g(x) = \pm \infty$
zu, so gilt dann \[ \lim\limits_{x \to a} \frac{f(x)}{g(x)} = \lim\limits_{x \to a} \frac{f'(x)}{g'(x)}\] sofern der rechte Limes existiert oder gegen $\pm \infty$ verläuft. Dabei ist egal, ob man $\lim\limits_{x \to a}$, $\lim\limits_{x \to a^+}$ oder $\lim\limits_{x \to a^-}$ betrachtet.
Betrachten wir wieder unsere letzte Funktion $\frac{e^x}{x}$. Zuerst müssen wir prüfen, ob wir einen der oben genannten Fälle haben. \[ \lim\limits_{x \to + \infty} \frac{e^x}{x} = \frac{\infty}{\infty}\] Da wir denn Fall haben, dass Zähler und Nenner gegen Unendlich gehen, können wir die Regel von de l'Hospital anwenden. \[ \lim\limits_{x \to + \infty} \frac{e^x}{x} = \lim\limits_{x \to + \infty} \frac{e^x}{1} = \infty\] Somit haben wir eine genauere Begründung für diesen Grenzfall.

Beweisidee

Zum Abschluss, wollen wir noch eine Beweisidee anhand von Fall 1) darstellen.
Es sei $a \in \mathbb{R}$, die beiden Funktionen $f$ und $g$ in $a$ differenzierbar und $g'(a) \ne 0$. Nun gilt die erste Aussage, also $f(a) = g(a) = 0$. Betrachten wir nun den Grenzwert \[ \lim\limits_{x \to a} \frac{f(x)}{g(x)} \] und subtrahieren im Zähler $f(a)$ und im Nenner $g(a)$.Dies verändert unsere Gleichung nicht, da beide Terme nach Annahme her Null sind. Somit ergibt sich: \[ \lim\limits_{x \to a} \frac{f(x)}{g(x)} = \lim\limits_{x \to a} \frac{f(x)-f(a)}{g(x)-g(a)} \] Nun erweitern wir mit $x-a \ne 0$ und formen etwas um. \begin{align} \lim\limits_{x \to a} \frac{f(x)-f(a)}{g(x)-g(a)} \cdot \frac{x-a}{x-a} &= \lim\limits_{x \to a} \frac{f(x)-f(a)}{g(x)-g(a)} \cdot \frac{\frac{1}{x-a}}{\frac{1}{x-a}} \\ &= \lim\limits_{x \to a} \frac{\frac{f(x)-f(a)}{x-a}}{\frac{g(x)-g(a)}{x-a}} \end{align} Betrachten wir aus Gründen der Übersicht als erstes den Zähler. \[ \lim\limits_{x \to a} \frac{f(x)-f(a)}{x-a}\] Wenn wir uns nun an den Differentialquotienten erinnern, so gibt der obige Ausdruck gerade die Steigung in $a$ an, also $f'(a)$. Im Nenner haben wir den gleichen Fall. Somit ergibt sich: \[ \lim\limits_{x \to a} \frac{f(x)}{g(x)} = \ldots =\lim\limits_{x \to a} \frac{\frac{f(x)-f(a)}{x-a}}{\frac{g(x)-g(a)}{x-a}} = \frac{f'(a)}{g'(a)}\] Der allgemeine Fall ist etwas schwerer zu beweisen.

Hinweis: Mithilfe der obigen Regeln, können wir nicht nur Grenzwerte der Fälle \[ \frac{\infty}{\infty} \quad \text{ und } \quad \frac{0}{0} \] versuchen zu berechnen, sondern zum Beispiel auch \[ 0 \cdot \infty.\] Nun fragen sich einige, wie geht denn das. Aber umformen ergibt: \[ 0 \cdot \infty = 0 \cdot \frac{1}{ \frac{1}{\infty}} = \frac{0}{\frac{1}{\infty}} = \frac{0}{0}\] Es sei gesagt, dass eine solche Rechnung, wie in der letzten Zeile keineswegs formal korrekt ist, aber es anschaulich erklärt, wie man auch andere Fälle auf einen der beiden genannten Fälle zurückführen kann.